题目列表

题目内容
The median of x , y , 8 and 11 is 19.

Quantity A

x

Quantity B

23


a , b , c and d are different positive numbers.

The average (arithmetic mean) of a and b is 30.

The average of a , b , c and d is 40.

Quantity A

The greatest possible value of d

Quantity B

99


Set A: {x, x, x, y, y, y, 3x+y, x–y }

If the median of set A is 10 and 0 < x < y, what is the range of set A?_____
In a certain set of numbers, 12.5 is 1.5 units of standard deviation above the mean, and 8.9 is 0.5 units of standard deviation below the mean. What is the mean of the set?

Give your answer to the nearest 0.1_____
Set X={a, b, c}, where a?< b < c. If the average (arithmetic mean) of a and b is 3x–13, and the average of b and c is 3x+11, what is the range of set X?_____
What is the average (arithmetic mean) of all multiples of 10 from 10 to 400 inclusive?
There are 10 employees in an office, not counting the office manager. The table shows how many employees have 0, 1, 2 or 3 pets. If the office manager also were included in the table, the average (arithmetic mean) number of pets per person would equal the median number of pets per person. How many pets does the office manager have? GRE、gre题库、gre模考、gre考满分
If the average (arithmetic mean) of five consecutive negative integers is 2k – 1, what is the difference between the greatest and least of the five integers?
Set X consists of 100 numbers. The average (arithmetic mean) of set X is 10, and the standard deviation is 4.6. Which of the following two numbers, when added to set X, will decrease the set`s standard deviation by the greatest amount?
If the average (arithmetic mean) of a and b is j, and the average of c, d, and e is k, what is the average of a, b, c, d, e and j ?
The average (arithmetic mean) of five numbers is 3x + 4. If one of the numbers is 7x - 4, what is the average of the other four numbers?
If the average (arithmetic mean) of seven consecutive integers is k + 2, then the product of the greatest and least integer is
The average (arithmetic mean) of 4 different integers is 75. If the largest integer is 90, what is the least possible value of the smallest integer?
Positive integers a, b, c, d and e are such that a < b < c < d < e. If the average (arithmetic mean) of the five numbers is 6 and d - b = 3, then what is the greatest possible range of the five numbers?
K = sum of the integers from 1 to 500 inclusive that are divisible by 5.

Quantity A

k

Quantity B

25,000


Yesterday, at a certain school, the ratio of boys to girls was 1 to 3. Today, an equal number of boys and girls joined the school. The number that joined was greater than zero and no students left.

Quantity A

Ratio of boys to girls now

Quantity B

$$\frac{1}{3}$$


In a group of 45 children, 60 percent of the children are boys, and 60 percent of the children are left-handed.

Quantity A

Number of boys who are left-handed

Quantity B

8


Clyde drove 30 miles in 20 minutes, and then drove an additional 10 miles in 10 minutes.

Quantity A

Clyde`s average speed for the entire trip.

Quantity B

75 miles per hour


Gene is 7 years older than Roberta.

6 years ago, Gene was twice as old as Roberta.

Quantity A

Roberta`s current age

Quantity B

12


In 12 years, Murray will be 4 times as old as he is now.

Quantity A

Number of years until Murray is 8 times as old as he is now

Quantity B

24


共收录:

25000 +道题目

5本备考书籍

最新提问